Középiskolai Matematikai és Fizikai Lapok
Informatika rovattal
Kiadja a MATFUND Alapítvány
Már regisztráltál?
Új vendég vagy?

Fórum: Érdekes matekfeladatok

  [1]    [2]    [3]    [4]    [5]    [6]    [7]    [8]    [9]    [10]    [11]    [12]    [13]    [14]    [15]    [16]    [17]    [18]    [19]    [20]    [21]    [22]    [23]    [24]    [25]    [26]    [27]    [28]    [29]    [30]    [31]    [32]    [33]    [34]    [35]    [36]    [37]    [38]    [39]    [40]    [41]    [42]    [43]    [44]    [45]    [46]    [47]    [48]    [49]    [50]    [51]    [52]    [53]    [54]    [55]    [56]    [57]    [58]    [59]    [60]    [61]    [62]    [63]    [64]    [65]    [66]    [67]    [68]    [69]    [70]    [71]    [72]    [73]    [74]    [75]    [76]    [77]    [78]    [79]    [80]    [81]    [82]    [83]    [84]    [85]    [86]    [87]    [88]    [89]    [90]    [91]    [92]    [93]    [94]    [95]    [96]    [97]    [98]    [99]    [100]    [101]    [102]    [103]    [104]    [105]    [106]    [107]    [108]    [109]    [110]    [111]    [112]    [113]    [114]    [115]    [116]    [117]    [118]    [119]    [120]    [121]    [122]    [123]    [124]    [125]    [126]    [127]    [128]    [129]    [130]    [131]    [132]    [133]    [134]    [135]    [136]    [137]    [138]    [139]    [140]    [141]    [142]    [143]    [144]    [145]    [146]    [147]    [148]    [149]    [150]    [151]    [152]    [153]    [154]    [155]    [156]    [157]    [158]    [159]    [160]    [161]  

Szeretnél hozzászólni? Jelentkezz be.
[2871] psbalint2009-01-26 17:02:35

megnézed azt a kis nyilacskás részt, és mindegyik elemhez hozzárendeled a tőle jobbra, három nyilacskára álló elemet :)

[2870] mmarta2009-01-26 08:37:05

Akkor már csak a vége nem világos, a háromszori hozzárendelés. Ezt hogyan kell?

Előzmény: [2869] S.Ákos, 2009-01-25 21:17:30
[2869] S.Ákos2009-01-25 21:17:30

63=12.5+3, szóval: P63=P60+3=(P5)12.P3=P3 A párosítás pedig abból adódott, hogy külön-külön megnézzük az egyes elemeknél, hogy P háromszori elvégzése mit rendel hozzá az adott elemhez.

Előzmény: [2868] mmarta, 2009-01-25 19:02:42
[2868] mmarta2009-01-25 19:02:42

Köszi! A P3 hogyan lett? És mi alapján állítjuk párba a végén?

Előzmény: [2867] S.Ákos, 2009-01-25 16:24:57
[2867] S.Ákos2009-01-25 16:24:57

Nézzük, hogy a permutáció melyik elemet melyikbe viszi: 1\to3\to5\to2\to4\to1. Ebből következik, hogy P5=1, mivel P ötszöri elvégzése identitás. Vagyis P^{63}=P^3=\left(\matrix{1&2&3&4&5\cr 2&3&4&5&1\cr}\right)

Előzmény: [2866] mmarta, 2009-01-25 11:53:47
[2866] mmarta2009-01-25 11:53:47

Sziasztok! Segítséget szeretnék én is kérni a köv. feladathoz.  P=\binom{12345}{34512} Adja meg a P63 permutációt. Egyszerű, szájbarágós magyarázat, megoldás kellene, hogy meg is értsem :)

Köszönöm

[2865] Kemény Legény2009-01-24 02:18:24

Hát, semmi sem elég jó Neked :)

Rendben, akkor az új "tétel": A\cos\alpha+B\cos\beta+C\cos\gamma\leq\frac{K}{2}

Az ezzel egyenértékű A\cos\beta+B\cos\gamma+C\cos\alpha\leq\frac{K}{2} és A\cos\gamma+B\cos\alpha+C\cos\beta\leq\frac{K}{2}

egyenlőtlenségek összege (A+B+C)(\cos\alpha+\cos\beta+\cos\gamma)\leq\frac{3}{2}K

Ismert, hogy \cos\alpha+\cos\beta+\cos\gamma\leq\frac{3}{2}

Tehát valamelyik állítás igaz, azaz a "szokásos érvvel" az összes is, a tételt "beláttuk".

Ezt mutatja nyilván az A=1,B=\sqrt2, C=\sqrt2, \alpha=90°, \beta=45°, \gamma=45° példa is, amikor A\cos\alpha+B\cos\beta+C\cos\gamma=\frac{\sqrt2}{2}\cdot2\sqrt2=2

Míg a jobb oldal \frac{K}{2}=(1+2\sqrt2)/2=0.5+\sqrt2<2

Amit csináltam az eredeti "bizonyítás" kapcsán, hogy lehagytam a 2-es kitevőket, és a koszinuszra cseréltem a kotangenst, és a jobb oldalra a kerület konstansszorosa került. Ha még mindig vannak ellenvetéseid (pl. szép-szép, de egy kontangenses ellenpélda még szebb lenne, sőt miért nem cáfolom meg rögtön az eredetileg kitűzött A-feladatot is), akkor szólj bátran.

Előzmény: [2864] rizsesz, 2009-01-23 23:47:26
[2864] rizsesz2009-01-23 23:47:26

Nálad mindkét változó-tömegnek van a jobb oldalon is függvénye (a+b+c=k, A+B+C=K), míg az én esetemben ctg\alpha, ctg\beta, ctg\gamma variálása nem befolyásolja a másik oldalt (T, amely csak A, B és C által meghatározott).

Előzmény: [2863] Kemény Legény, 2009-01-23 17:45:05
[2863] Kemény Legény2009-01-23 17:45:05

Ha csak az kell, tudok én egyenértékű állításokból is ellenpéldát mutatni. Legyenek 2 háromszög oldalai A,B,C és a,b,c, kerületeik K,k. Állítás: Aa+Bb+Cc\geq \frac{Kk}{3} Egyenértékű ezzel: Ab+Bc+Ca\geq \frac{Kk}{3} Szintén egyenértékű: Ac+Ba+Cb\geq\frac{Kk}{3}

Összeadva a 3 egyenlőtlenséget: a bal oldal (A+B+C)(a+b+c)=Kk azaz valamelyikük igaz, ekkor az "elv" miatt mindhárom igaz.

Ellenpélda: (A=2,B=2,C=3,a=1,b=1,c=0.1,K=7,k=2.1,Aa+Bb+Cc=2+2+0.3=4.3<4.9=Kk/3)

Előzmény: [2862] rizsesz, 2009-01-23 10:55:57
[2862] rizsesz2009-01-23 10:55:57

Én értem ezt a logikai utat. Pontosan ezért kérdeztem rá. Jelen esetben azonban ha akármelyik igaz, az egyenértékű az állítással.

Ezzel szemben az általad adott példában a 3 állítás nem egyenértékű.

Azt is értem, hogy az eredeti állításból nem juthatunk el a + 2 általam felírt egyenlőtlenséghez.

Szóval már majdnem meggyőztél, de még kötözködöm :)

Előzmény: [2861] Kemény Legény, 2009-01-23 10:11:10
[2861] Kemény Legény2009-01-23 10:11:10

Az eredeti "bizonyítás" során a 3 állítás bármelyikének igazolása önmagában elegendő lenne a teljes megoldáshoz, de onnantól kezdve, hogy mindhármat felírtuk egyszerre, akkor már csak azt tudjuk belátni, hogy minden háromszögben valamelyik igaz. Abból pedig, hogy egy állítás egy tetszőleges háromszögben igaz az oldalak/szögek valamilyen sorrendjére, nem következik, hogy minden háromszögben az oldalak/szögek tetszőleges sorrendjére igaz lenne.

Az állítások felírásában valóban nincs kitüntetett szerepe semminek, de amikor találunk egyet a 3 közül, ami igaz, abban már az oldalak/szögek egy kitüntetett sorrendben szerepelnek, és épp ez veszi el az általánosítás lehetőségét.

Előzmény: [2856] rizsesz, 2009-01-23 02:06:57
[2860] rizsesz2009-01-23 09:14:59

Ugyanis ha már az egyik igaz, akkor mivel a szorzók nincsenek meghatározva, így bármilyen formában odapakolhatod őket az oldalak mellé együtthatónak.

Szóval szerintem az én logikám nem alkalmazható az a+b+4c-s példára.

Előzmény: [2858] Valezius, 2009-01-23 06:41:45
[2859] rizsesz2009-01-23 08:59:53

Értem az indoklást, de mégsem :) mármint abba már belenyugodtam, hogy nem jó a bizonyítás, de!

Itt azért jóval többről van szó, mint a ti példátokban! Az állítás, mármint átalakítottam a feladatbelit, azt mondja ki, hogy bármilyen szög-kombinációban igaz ez az állítás. Míg a ti esetekben ti pontosan kihasználjátok, hogy éppen melyik oldalnak adtok 4-es szorzót, és így születik meg az ellenpélda.

Én azt hiszem, hogy mégiscsak jó a bizonyításom.

Előzmény: [2858] Valezius, 2009-01-23 06:41:45
[2858] Valezius2009-01-23 06:41:45

Szép próbálkozás :)

3-ból 1igaz, akkor hiába rendezed át a számokat továbbra is 3-ból 1 lesz igaz, csak egy másik.

pl. a=0, b=3, c=3

a+b>=4, a+c>=4, a+b>=4

persze ha teljesül, hogy a+b+c>=6, akkor legalább az egyik igaz. De ha átrendezed a számokat, akkor továbbra is csak 1marad igaz, nem mind a három.

A lényeg: Abból, hogy felcserélhetők a számok csak az következik, hogy feltehető, hogy a+b>=4. Semmi más. (Mivel ha a+b<4, akkor átrendezem a számokat)

Utolsó kérdésre: 3állatás igazságából következik a negyedik, de fordítva nyilván nem, ha látom, hogy teljesül a 4. akkor mitől lenne igaz az első 3mindegyike? Most erre biztos lehet nagyon primitív példákat gyártani, de remélem így is érted :)

Előzmény: [2856] rizsesz, 2009-01-23 02:06:57
[2856] rizsesz2009-01-23 02:06:57

Igazából pontosan jól látod a problémámat - akkor viszont hol bukik el a dolog, azaz miért nincsen kitüntetett szerepe a másik háromszög szögeinek? (a csak akkort most érzem :) tehát ha az eredeti állítás igaz, akkor a másik kettő is, így az összegük is, viszont az a kérdésem, hogy min múlik a visszafelé?)

Előzmény: [2855] Kemény Legény, 2009-01-22 22:58:59
[2855] Kemény Legény2009-01-22 22:58:59

Ha jó a gondolatmenet, akkor pl. bizonyítható, hogy "minden háromszögben a+b+4c>=2K", hiszen az ehhez hasonló a+4b+c>=2K és 4a+b+c>=2K egyenlőtlenségek összege 6(a+b+c)=6K, azaz vmelyik a 3 egyenlőtlenség közül igaz kell legyen és akkor az "elv" miatt az összes többi is.

/Holott pl. az a=1,b=1,c=0.001 oldalú háromszögben nem igaz az állitás/

Előzmény: [2857] rizsesz, 2009-01-22 15:05:23
[2857] rizsesz2009-01-22 15:05:23

Szerintetek az A.468-ra jó az alábbi megoldás:

A bal oldalon a tagokat az alábbi logika szerint átalakítva: A2*(-a2+b2+c2) - a zárójelben található kifejezés a cos-tétel miatt alakba A2*2bc*cos\alpha írható, ahonnan t=\frac{bcsin\alpha}{2} miatt bc=\frac{2t}{sin\alpha}. Ezt beírva kiemelhető t, ha ezt a logikát követjük mindhárom tagra behelyettesíthető ctg\alpha, és végül 4t-vel osztva azt kapjuk, hogy az állítás az A2*ctg\alpha+B2*ctg\beta+C2*ctg\gamma>=4T

Tekintsük az alábbi 2 további állítást:

A2*ctg\beta+B2*ctg\gamma+C2*ctg\alpha>=4T

A2*ctg\gamma+B2*ctg\alpha+C2*ctg\beta>=4T

Ha ezek közül az egyik teljesül, akkor a másik kettő is, hiszen \alpha, \beta, \gamma között annyi az összefüggés, hogy összegük 180 fok, más megkötést pedig nem generálnak az egyenlőtlenségekben.

Ha tehát ezen 3 összefüggés összege igaz, akkor legalább az egyik igaz (különben indirekten következne, hogy nem teljesülhet az összegként létrejövő egyenlőtlenség).

Viszont ha az egyik igaz, akkor abból logikailag következik, hogy mindhárom igaz, hiszen csak a szögeket kell átrendezni, amit megtehetünk.

A 3 egyenlőtlenség összege: (ctg\alpha+ctg\beta+ctg\gamma)*(A2+B2+C2)>=12T

Ez pedig igaz, mert az első a Jensen-egyenlőtlenség miatt: (ctg\alpha+ctg\beta+ctg\gamma)>=3*(ctg(\alpha+\beta+\gamma)/3)=\sqrt{3}

(A2+B2+C2)>=4*\sqrt{3}*T pedig ismert.

Ez szerintetek jó?

[2854] Fálesz Mihály2009-01-22 13:46:42

TeX minitanfolyam

Előzmény: [2853] janomo, 2009-01-22 12:14:12
[2853] janomo2009-01-22 12:14:12

Általánosan egy akármilyen q(x) eleme Z[x] polinomra kilehet számolni könnyen a legendre(q(x)) összegeit, ha x végig megy az összes maradékon.

A q(x) a (p-1)/2-en ek összegét vesszük, akkor olyan típusú összegeket kell vizsgálni, hogy (1) k +2 a k-o +....+(p-1) k és egy ilyenről belehet bizonyítani, hogypontosan akkor nem 0, ha k (p-1) többszöröse. Ha pedig k többszöröse (p-1)-nek, akkor ez az összeg (-1)-et ad a kis Fermat tétel miatt

Tehát a p(x) (p-1)/2 polinomban össze kell gyűjteni azokat a tagokat, amelyek kitevője osztható (p-1)-el és ezeknek a tagoknak a együtthatóinak az össszege.

Ezt nehéznek tűnik kiszámolni, de valójában nem az, például a p(x)= x köb +1 polinomra szép eredmény jön ki. Az összeg maradéka -(p-1)/2 alatt a (p-1)/3 plusz 1.

Érdekes még, hogyha veszünk egy akármilyen polinomot, akkor ezzel a polinomos módszerrel kihozható, hogy létezik egy c konstans melyre minden p-re az összeg maradéka c*sqrt(p)-nél közelebb van a 0-hoz. Ez azt jelenti, hogy egy polinom helyettesítési értékeiközt is nagyjából egyenletesen oszlanak el a kvadratikus és nem kvadratikus maradékok.

Érdekes kérdés, hogymely q(x) polinomra 0 ez az összeg minden p prímre.

[2852] nadorp2009-01-21 09:17:29

Ha a balodali összeget S-nek jelöljük, akkor |S|\leqp-2 miatt elég bizonyítani, hogy S\equiv-1 mod(p), vagy ami ugyanaz, a mod p testben S=-1.

Felhasználva, hogy \left(\frac ap\right)=a^{\frac{p-1}2} és hogy \left(\frac{ab}p\right)=\left(\frac ap\right)\left(\frac bp\right)=\frac{\left(\frac ap\right)}{\left(\frac bp\right)}

S=\sum_{n=1}^{p-2}\frac{{\left(n+1\right)}^{\frac{p-1}2}}{n^{\frac{p-1}2}}=\sum_{n=1}^{p-2}\left({1+\frac1n}\right)^{\frac{p-1}2}=\sum_{n=2}^{p-1}n^{\frac{p-1}2}=\sum_{n=2}^{p-1}\left(\frac np\right)=-1

Az utolsó lépésekben felhasználtuk, hogy 1+\frac1n a 2,3,...(p-1) értékeket veszi fel valamilyen sorrendben és mindegyiket egyszer, másrészt hogy ugyanannyi négyzetes maradék van, mint négyzetes nemmaradék.

Előzmény: [2849] psbalint, 2009-01-20 20:16:52
[2851] psbalint2009-01-20 20:47:44

igen azon csakúgy átugrottam hogy p eleme P-nek, de valóban :)

[2850] nadorp2009-01-20 20:28:32

Lagrange = Legendre

p páratlan prím

Előzmény: [2849] psbalint, 2009-01-20 20:16:52
[2849] psbalint2009-01-20 20:16:52

üdv. van még egy szép, érdekes és megoldatlan, lagrange-szimbólumos feladatom :)

340. feladat bizonyítsuk be, hogy ha 'p' páratlan, akkor:

[2848] jenei.attila2009-01-18 11:46:07

Szép gondolat, érdemes lenne kidolgozni. Apróbb pontosításokat engedj meg. A te jelöléseidet megtartva, jelöljük \phi-t \phia-val, jelezve, hogy ennek az operátornak a a paramétere. A \phia ekkor így néz ki:

\phia:F\toF

\phia(h)=x\mapstoh(x+a)-h(x)

(nyilván elírtad, amikor a helyett p-t írtál). Az valóban elég lenne a h előállíthatóságához (a és b-periodikus fv-ek összegeként) ha, \phia(Q)=Q teljesülne. Azonban nem biztos, hogy ez szükséges is, mert mi van ha Q-nak létezik olyan eleme, amely nem áll elő egyetlen F-beli elem \phia szerinti képeként sem. Ehelyett azt kellene bizonyítani, hogy ha \phia(h)\inQ, akkor \phia(h)\in\phi(Q) is teljesül.Mindez persze csak akkor, ha a és b nem összemérhetők. Ez utóbbi (egyébként szükséges) feltételt még nem használtuk ki.

Előzmény: [2847] Tibixe, 2009-01-17 16:54:58
[2847] Tibixe2009-01-17 16:54:58

Ebben a legutóbbiban igazad van, még egyszer elolvasva ez éppen a kiválasztási axióma által létrehozott borzalmak közé tartozik.

Gondolkodtam rajta, hogy csoportokkal esetleg szebben megfogalmazható, de végül nem sikerült bizonyítanom . Azért beírom, hogy meddig jutottam.

F legyen a valósakon értelmezett függvények csoportja ( összeadásra )

P legyen az a-periodikus függvények csoportja

Q legyen az b-periodikus függvények csoportja

Tehát P és Q F részcsoportjai.

Az egyik állítás úgy néz ki, hogy h felírható P-beli és Q-beli elemek összegeként, azaz

h=p+q    p\inP  q\inQ

A másikat megfogalmazni viszont kicsit bonyolultabb.

Vegyük a

\phi:F\toF

\phi(h)=x\mapstoh(x+p)-h(x)

függvényt. Szemléletesen: \phi megadja, hogy egy függvény hogyan nem a szerint periodikus. Ha ez ,,nemperiodikusság'' b szerint periodikus, akkor benne lesz Q-ban. Így megfogalmazva a második állítás:

\phi(h)\inQ

Tehát bizonyítandó, hogy:

h=p+q  p\inP  q\inQ    \leftarrow\rightarrow    \phi(h)\inQ

\phi művelettartó, tehát F-nek egy endomorfizmusa, mégpedig olyan, ami az egész P részcsoportot a nullába képezi. Ebből még következik az is, hogy \phi képe F/P-vel izomorf.

A balról jobbra irány bizonyítása: A jobb oldali képletbe írjuk be, hogy h=p+q és alkalmazzuk \phi művelettartását:

\phi(p+q)\inQ

\phi(p)+\phi(q)\inQ

\phi(p)-ről tudjuk, hogy nulla.

\phi(q)\inQ

\phi(q) pedig két Q-beli függvény különbsége, tehát maga is Q-beli.

A jobbról balra irány bizonyítása: Ha

\phi(q)=\phi(h)

teljesül valamilyen Q-beli q-ra, akkor q és h csak egy P-beli elemmel különbözhetnek. Innen még azt kéne bizonyítani, hogy \phi(Q)=Q, tehát hogy \phi Q-ra való megszorítása szürjektív és készen lennénk.

A Wikipedia ír egy kiválasztásai axiómával ekvivalens feltételt a szürjektivitásra itt: http://en.wikipedia.org/wiki/Surjective Valószínűleg ezt kéne valahogy okosan használni és úgy ezt a gondolatmenetet is be lehetne fejezni.

Nekem viszont csak eddig tartott a tehetségem meg az energiám.

  [1]    [2]    [3]    [4]    [5]    [6]    [7]    [8]    [9]    [10]    [11]    [12]    [13]    [14]    [15]    [16]    [17]    [18]    [19]    [20]    [21]    [22]    [23]    [24]    [25]    [26]    [27]    [28]    [29]    [30]    [31]    [32]    [33]    [34]    [35]    [36]    [37]    [38]    [39]    [40]    [41]    [42]    [43]    [44]    [45]    [46]    [47]    [48]    [49]    [50]    [51]    [52]    [53]    [54]    [55]    [56]    [57]    [58]    [59]    [60]    [61]    [62]    [63]    [64]    [65]    [66]    [67]    [68]    [69]    [70]    [71]    [72]    [73]    [74]    [75]    [76]    [77]    [78]    [79]    [80]    [81]    [82]    [83]    [84]    [85]    [86]    [87]    [88]    [89]    [90]    [91]    [92]    [93]    [94]    [95]    [96]    [97]    [98]    [99]    [100]    [101]    [102]    [103]    [104]    [105]    [106]    [107]    [108]    [109]    [110]    [111]    [112]    [113]    [114]    [115]    [116]    [117]    [118]    [119]    [120]    [121]    [122]    [123]    [124]    [125]    [126]    [127]    [128]    [129]    [130]    [131]    [132]    [133]    [134]    [135]    [136]    [137]    [138]    [139]    [140]    [141]    [142]    [143]    [144]    [145]    [146]    [147]    [148]    [149]    [150]    [151]    [152]    [153]    [154]    [155]    [156]    [157]    [158]    [159]    [160]    [161]